The domain on which a function is continuous

But I guess I should be looking at the limit as x approaches -1 and y approaches 0, which is the same as the first function I posted, but there's a difference between the x approach limit and the y approach limit right?To make things easier, can I just substitute x=0 into the first function i posted?Because I'm pretty sure that's the first function I posted just with x=0 substituted in which means that the limit as (x,y) approaches (0,0) is 0 right?I'm having trouble understanding why we have to approach (-1,0) along different lines to know if the function is continuous?I can't test x approaches -1, or
  • #1
mrcleanhands

Homework Statement



Determine domain on which the following function is continuous
[tex]f(x,y)= \left\{\begin{array}{cc}
\frac{x(x+1)y^{2}}{(x+1)^{2}+y^{2}} & (x,y)\neq(-1,0)\\
1 & (x,y)=(-1,0)
\end{array}\right.[/tex]

Homework Equations





The Attempt at a Solution



Because the numerator component is a rational function it will be continuous for all [itex](x,y)\in\mathbb{R}[/itex] except on (x,y)=(-1,0) and since the denominator has been set a domain of a single point (x,y)=(-1,0), it is also continuous in that domain. To test whether the whole piece-wise function is continuous we should look for the limits as [itex](x,y)\rightarrow(-1,0)[/itex].
[tex]\underset{(x,y)\rightarrow(-1,0)}{\lim}\frac{x(x+1)y^{2}}{(x+1)^{2}+y^{2}}[/tex] using the path y=mx
[tex]\underset{(x,y)\rightarrow(-1,-m)}{\lim}\frac{-1(-1+1)y^{2}}{m^{2}}=0[/tex]

Approaching (-1,0) from the line y=mx gives us a limit of 0 which contradicts the limit of 1 for the 2nd component in the piece-wise function. Therefore the function as a whole is continuous for all [itex](x,y)\in\mathbb{R}[/itex] except on (x,y)=(-1,0).

I feel like something is not right with my reasoning or explanation. The function on the whole is continuous because it's defined for every x,y for all real numbers.
 
Last edited by a moderator:
Physics news on Phys.org
  • #2
Your method is correct: the limit depends on how you approach the point (-1,0). A function defined for all (x,y) is not necessarily continuous everywhere - why should it be? Think of the following 1D example: f(x)=1/x forall x, f(0)=0.
 
  • #3
mrcleanhands said:

Homework Statement



Determine domain on which the following function is continuous
$$f(x,y)=\begin{array}{cc}
\frac{x(x+1)y^{2}}{(x+1)^{2}+y^{2}} & (x,y)\neq(-1,0)\\
1 & (x,y)=(-1,0)
\end{array}$$

Homework Equations



The Attempt at a Solution



Because the numerator component is a rational function it will be continuous for all ##(x,y)\in\mathbb{R}## except on (x,y)=(-1,0)
and since the denominator has been set a domain of a single point (x,y)=(-1,0)
it is also continuous in that domain.
That all doesn't make sense.

The denominator of [itex]\displaystyle \ \frac{x(x+1)y^{2}}{(x+1)^{2}+y^{2}}\ [/itex] is zero only at (x, y) = (-1, 0) . Therefore, we know that f(x,y) is continuous everywhere, except possibly at (x, y) = (-1, 0) .
To test whether the whole piece-wise function is continuous we should look for the limits as ##(x,y)\rightarrow(-1,0)##
$$\underset{(x,y)\rightarrow(-1,0)}{\lim}\frac{x(x+1)y^{2}}{(x+1)^{2}+y^{2}}$$
using the path y=mx
$$\underset{(x,y)\rightarrow(-1,-m)}{\lim}\frac{-1(-1+1)y^{2}}{m^{2}}=0$$
Approaching (-1,0)
from the line y=mx gives us a limit of 0 which contradicts the limit of 1 for the 2nd component in the piece-wise function. Therefore the function as a whole is continuous for all ##(x,y)\in\mathbb{R}##
except on (x,y)=(-1,0)

I feel like something is not right with my reasoning or explanation. The function on the whole is continuous because it's defined for every x,y for all real numbers.
The line y = mx, doesn't pass through (-1,0) , does it ?


Use the following for LaTeX renderings.
Code:
For inline LaTeX use:
[itex] [/itex]    or    ## ##

For stand alone, \displaystyle use:
[tex] [/tex]    or     $$ $$
 
  • #4
yeah, you're right. so how can I find the limit for the second function then? There just isn't a limit?
 
  • #5
mrcleanhands said:
yeah, you're right. so how can I find the limit for the second function then? There just isn't a limit?

What second function ?
 
  • #6
I think I realized what was wrong with it. If I can't approach (-1,0) from the line y=mx then I can't approach it from the x axis, y axis, or z axis right?

So if I want to find the limit of this function
[itex]\underset{(x,y)\rightarrow(-1,0)}{\lim}\frac{x(x+1)y^{2}}{(x+1)^{2}+y^{2}}[/itex]

I cannot test a line approaching (-1,0) by letting y=x as I can when looking for the limit when we approach (0,0).

Does this mean then that I should change the function to find the limit along different lines?

E.g. substitute (x-1) into x and then find the limit as (x,y) --> (0,0) will be the equivalent.

So

[tex]\underset{(x,y)\rightarrow(-1,0)}{\lim}\frac{x(x+1)y^{2}}{(x+1)^{2}+y^{2}}
=\underset{(x,y)\rightarrow(0,0)}{\lim}\frac{(x-1)xy^{2}}{x{}^{2}+y^{2}}
[/tex]
right?
 
  • #7
mrcleanhands said:
So

[tex]\underset{(x,y)\rightarrow(-1,0)}{\lim}\frac{x(x+1)y^{2}}{(x+1)^{2}+y^{2}}
=\underset{(x,y)\rightarrow(0,0)}{\lim}\frac{(x-1)xy^{2}}{x{}^{2}+y^{2}}
[/tex]
right?
Yes, this is correct.

Hint: consider applying the inequality ##|xy| \leq \frac{1}{2}(x^2 + y^2)## to
$$\left| \frac{(x-1)xy^2}{x^2 + y^2} \right|$$
 
  • #8
jbunniii said:
Yes, this is correct.

Hint: consider applying the inequality ##|xy| \leq \frac{1}{2}(x^2 + y^2)## to
$$\left| \frac{(x-1)xy^2}{x^2 + y^2} \right|$$

How did you get that inequality?

Also what do you meany by applying it to $$\left| \frac{(x-1)xy^2}{x^2 + y^2} \right|$$
 
  • #9
For any x, y, [itex](x+ y)^2= x^2+ 2xy+ y^2\ge 0[/itex] so [itex]-2xy\le x^2+ y^2[/itex] and [itex](x- y)^2= x^2- 2xy+ y^2\ge 0[/itex] so [itex]2xy\le x^2+ y^2[/itex].

So
[tex]\left|\frac{(x- 1)xy^2}{x^2+ y^2}\right|=\left|(x- 1)y\frac{xy}{x^2+ y^2}\right|\le\frac{1}{2}\left|(x-1)y\right|[/tex]
 
  • #10
And this is related to epsilon delta proofing right? (struggling with at at the moment so can't really connect this)...

Btw I'm not supposed to use epsilon delta proofing to test the limit in this question. Just supposed to approach it from different lines which I haven't succeeded in doing thus far.
 
  • #11
OK, going back to your original expression:
$$\frac{x(x+1)y^{2}}{(x+1)^{2}+y^{2}}$$
what limit do you get if you approach ##(-1,0)## along the line ##y = 0##?
 
  • #12
Removed
 
Last edited:
  • #13
jbunniii said:
OK, going back to your original expression:
$$\frac{x(x+1)y^{2}}{(x+1)^{2}+y^{2}}$$
what limit do you get if you approach ##(-1,0)## along the line ##y = 0##?

[itex]f(x,0)=0[/itex]
 
  • #14
mrcleanhands said:
[itex]f(x,0)=0[/itex]
So what can you conclude?
 
  • #15
I didn't think you could conclude anything from that apart from what it is, approaching (-1,0) along the y-axis produces a limit of 0 but that doesn't mean it's true for the whole function etc
 
  • #16
mrcleanhands said:
I didn't think you could conclude anything from that apart from what it is, approaching (-1,0) along the y-axis produces a limit of 0 but that doesn't mean it's true for the whole function etc
If the function was continuous at (-1,0), then the limit would have to be the same, no matter what direction you approach from. And what would the limit have to equal?
 
  • #17
jbunniii said:
If the function was continuous at (-1,0), then the limit would have to be the same, no matter what direction you approach from. And what would the limit have to equal?


0, same as approaching (-1,0) from the line y=0
 
  • #18
mrcleanhands said:
0, same as approaching (-1,0) from the line y=0
No, look at how the function is defined at (-1,0). What would the limit as (x,y) approaches (-1,0) have to be in order for the function to be continuous?
 
  • #19
I'm not following :(
What do you mean what would the limit have to be for the function to be continuous?

For the function to be continuous the limit would have to be the same as (x,y) approaches (-1,0) from any directional line...
 
  • #20
mrcleanhands said:
I'm not following :(
What do you mean what would the limit have to be for the function to be continuous?

For the function to be continuous the limit would have to be the same as (x,y) approaches (-1,0) from any directional line...
What is the value of the function at (-1,0)? Doesn't the limit have to match that value if the function is continuous?
 
  • #21
Ok I see that the limit would have to match the function. It's undefined at (-1,0).

But that does not mean that the limit does not exist right? since the function may get infinitely close to some point but not touch it.
 
  • #22
mrcleanhands said:
Ok I see that the limit would have to match the function. It's undefined at (-1,0).

But that does not mean that the limit does not exist right? since the function may get infinitely close to some point but not touch it.
The piecewise defined function, f(x,y), which appears in the OP is defined everywhere, even at (-1,0). In fact, f(-1,0) = 1 .

The expression, [itex]\ \displaystyle \frac{x(x+1)y^{2}}{(x+1)^{2}+y^{2}}\ [/itex] is defined for all x & y, except at the point, (-1,0). And the limit of this expression as (x,y) → (-1,0) does exist -- but you have not proved that. This limit needs to be equal to f(-1,0) , if f is to be continuous at (-1,0) .
 
  • #23
Okay I follow that. I get one of the limits as (x,mx) --> 0 as 0 so that means the function is not continuous.

However, it's weird because the question asks me to show that the limit exists or doesn't and support my claim without epsilon delta proofing. I keep getting 0 when approaching it from different lines but how do I ever know if there isn't some weird line which when approaching (0,0) doesn't = 0. I don't see how I can answer this question without epsilon delta unless I find one line which doesn't approach 0 - showing the function has no limit...
 
  • #24
mrcleanhands said:
Okay I follow that. I get one of the limits as (x,mx) --> 0 as 0 so that means the function is not continuous.

However, it's weird because the question asks me to show that the limit exists or doesn't and support my claim without epsilon delta proofing. I keep getting 0 when approaching it from different lines but how do I ever know if there isn't some weird line which when approaching (0,0) doesn't = 0. I don't see how I can answer this question without epsilon delta unless I find one line which doesn't approach 0 - showing the function has no limit...
The following link discusses finding the same limit for this very same expression.

https://www.physicsforums.com/showthread.php?t=683042

See if you can follow it. The basic idea was to shift the function one unit to the right,
then change to polar coordinates and find the limit as r → 0 .
 
  • #25
I did ... dammit and it took me a few minutes to answer the question on 3 lines where as I spent something like 30min+ figuring it out before with tons of lines and arbitrary stabs.
 
  • #26
jbunniii said:
Yes, this is correct.

Hint: consider applying the inequality ##|xy| \leq \frac{1}{2}(x^2 + y^2)## to
$$\left| \frac{(x-1)xy^2}{x^2 + y^2} \right|$$

I'm still curious why this was brought up however...
 
  • #27
mrcleanhands said:
jbunniii said:
Yes, this is correct.

Hint: consider applying the inequality ##|xy| \leq \frac{1}{2}(x^2 + y^2)## to
$$\left| \frac{(x-1)xy^2}{x^2 + y^2} \right|$$

I'm still curious why this was brought up however...

It's one way to attack the problem, although it does appear most suitable for an ε-δ proof . Notice that x2 + y2 = r2 .

In general, to find a limit as (x,y) → (0,0), using polar coordinates is very useful. You then have a limit in one variable: r → 0 .
 
  • #28
mrcleanhands said:
I'm still curious why this was brought up however...

jbunniii said:
Yes, this is correct.

Hint: consider applying the inequality ##|xy| \leq \frac{1}{2}(x^2 + y^2)## to
$$\left| \frac{(x-1)xy^2}{x^2 + y^2} \right|$$

This allows you to easily see that
$$\lim_{(x,y) \to (0,0)} \frac{(x-1)xy^{2}}{x^{2}+y^{2}} = 0$$
To see this, we note that
$$\begin{align}
\left| \frac{(x-1)xy^{2}}{x^{2}+y^{2}} \right| &=
\left| \frac{(x-1)y}{x^2 + y^2} \right| |xy| \\
&\leq \left| \frac{(x-1)y}{x^2 + y^2} \right| \left|\frac{1}{2}(x^2 + y^2)\right| \\
&= \frac{1}{2} \left| (x-1)y\right| \\
&\leq \frac{1}{2} |x-1| |y|
\end{align}$$
and the final expression clearly approaches ##0## as ##(x,y) \to (0,0)##.
 
  • #29
Ok, got it. Thank you!

This forum is awesome.

:D
 

1. What is the definition of continuity for a function?

The domain on which a function is continuous is the set of values for which the function is defined and has no abrupt changes or breaks. In other words, the function can be drawn without lifting the pen from the paper.

2. How can I determine if a function is continuous on a given domain?

To determine if a function is continuous on a given domain, you can use the three-part definition of continuity. This means that the function must be defined at every point in the domain, the limit of the function as x approaches a point in the domain must exist, and the limit must be equal to the value of the function at that point.

3. Can a function be continuous on one domain and not on another?

Yes, a function can be continuous on one domain and not on another. This is because the definition of continuity only applies to a specific domain and does not guarantee continuity on all possible domains.

4. What are some common types of functions that are continuous on their domain?

Some common types of functions that are continuous on their domain include polynomials, exponential functions, trigonometric functions, and logarithmic functions.

5. How is continuity related to differentiability?

Continuity is a necessary but not sufficient condition for differentiability. This means that a function must be continuous on an interval in order to be differentiable on that interval, but a function can be continuous without being differentiable.

Similar threads

  • Calculus and Beyond Homework Help
Replies
10
Views
824
  • Calculus and Beyond Homework Help
Replies
27
Views
730
  • Calculus and Beyond Homework Help
Replies
5
Views
216
  • Calculus and Beyond Homework Help
Replies
5
Views
1K
  • Calculus and Beyond Homework Help
Replies
3
Views
264
  • Calculus and Beyond Homework Help
Replies
3
Views
516
  • Calculus and Beyond Homework Help
Replies
6
Views
998
  • Calculus and Beyond Homework Help
Replies
3
Views
809
  • Calculus and Beyond Homework Help
Replies
19
Views
940
  • Calculus and Beyond Homework Help
Replies
6
Views
1K
Back
Top